Respuesta modificada. 2017-07-01
No hay contradicción porque tu análisis solo incluye lo que le sucede a la sustancia de trabajo gaseosa en el motor Stirling, y descuida un componente crucial del motor llamado regenerador. Si el regenerador no se incluye como un componente del motor cuando realizamos el análisis de eficiencia, entonces no tenemos un dispositivo que califique como un motor de calor operando entre dos temperaturas, y por lo tanto no deberíamos esperar que se ajuste al Teorema de Carnot como lo declaré en la versión original de esta respuesta.
Sin embargo, si tomamos en cuenta correctamente el regenerador, entonces encontramos que la eficiencia del motor es la eficiencia de Carnot.
Por supuesto, todo el análisis aquí es idealizado en el cual asumimos, por ejemplo, que no hay pérdidas de energía debido a la fricción en los componentes del motor.
Detalles.
Un motor stirling es más complejo de lo que el diagrama $P$-$V$ dibujado en la declaración de la pregunta parece indicar. Si reducimos conceptualmente el motor a su forma más simple, contiene dos componentes fundamentales:
- Una sustancia de trabajo gaseosa . Este es la parte del motor cuyo estado termodinámico viaja a lo largo de la curva en el diagrama $P$-$V$.
3. Un regenerador. Esta parte del motor absorbe y almacena la energía cedida por la sustancia de trabajo gaseosa mediante transferencia de calor durante el proceso $2\to 3$ y luego devuelve esa misma energía a la sustancia de trabajo gaseosa durante el proceso $4\to 1$.
El punto crucial es que cuando se incluye el regenerador, no hay transferencia neta de calor dentro o fuera del motor durante los procesos $2\to 3$ y $4\to 1$. La energía que abandona la sustancia de trabajo gaseosa durante el proceso $2\to 3$ por transferencia de calor se almacena en el regenerador, y ese calor se devuelve a la sustancia de trabajo durante el proceso $4\to 1$. No se transfiere calor entre el motor y su entorno durante estas etapas del ciclo.
Se sigue que el único calor transferido al motor en su totalidad se transfiere durante $1\to 2$. Esto califica al dispositivo como un motor de calor (ver respuesta antigua a continuación) y la eficiencia del motor se calcula entonces como la relación de la salida de trabajo neto dividido por la entrada de calor en el proceso $1\to 2$. Esto produce la eficiencia de Carnot como debería.
Mi respuesta original afirmaba que el ciclo dibujado no representa la operación de un motor de calor que opera entre dos temperaturas, pero estaba descuidando el regenerador, y creo que esto es lo que implícitamente hiciste en el cálculo que realizaste originalmente también, y esto produjo una eficiencia incorrecta.
Respuesta original, incompleta.
No hay contradicción. El ciclo Stirling que dibujaste arriba es reversible pero no opera entre dos depósitos a temperaturas fijas $T_1$ y $T_2$. Las partes isovolumétricas del ciclo operan a temperaturas cambiantes continuamente (piensa en la ley de los gases ideales).
Añadido Antiguo. Nota que en termodinámica, se dice que un motor de calor opera (o trabaja) entre (dos depósitos a) temperaturas $T_1$ y $T_2$ siempre y cuando todo el calor que absorbe o cede se haga a una de esas dos temperaturas.
Para darle credibilidad a esta definición (que es esencialmente implícita en la mayoría de las discusiones de motores de calor que he visto), aquí hay una cita del texto de termodinámica de Fermi:
En la sección anterior describimos un motor cíclico reversible, el motor de Carnot, que realiza una cantidad de trabajo $L$ durante cada uno de sus ciclos al absorber una cantidad de calor $Q_2$ de una fuente a la temperatura $t_2$ y cediendo una cantidad de calor $Q_1$ a una fuente a la temperatura más baja $t_1$. Diremos que dicho motor trabaja entre las temperaturas $t_1$ y $t_2.
0 votos
Estás calculando mal la eficiencia. Para un ciclo de motor arbitrario, tienes $Q_{H} = Q_{L} + W$ y $e = \frac{W}{Q_{H}}$. No estás dividiendo por tu calor de entrada.
0 votos
He editado la pregunta para claridad sobre lo que estoy calculando. El denominador en la eficiencia debería ser el calor que se introduce en el motor. La forma en que calculo el calor es aquel que es $> 0$. ¿Estás de acuerdo?
2 votos
Relacionado con la cuestión de la reversibilidad de las curvas en el espacio de estado termodinámico: physics.stackexchange.com/questions/78405/…